Posts Tagged ‘Vandermonde matrix’

One of the determinants that appear a lot in mathematics is the so-called Vandermonde determinant.

Definition. The n \times n Vandermonde matrix is defined as follows

\displaystyle V(x_1, \cdots , x_n):=\begin{pmatrix}1 & x_1 & \cdots & x_1^{n-1} \\ 1 & x_2 & \cdots & x_2^{n-1} \\ \vdots & \vdots & \cdots & \vdots \\ 1 & x_n & \cdots & x_n^{n-1}\end{pmatrix}.

The determinant of the matrix V(x_1, \cdots ,x_n) is called the Vandermonde determinant.

Of course, the value of \det V(x_1, \cdots , x_n) is quite well-known but let’s find it anyway.

Remark 1. Expanding along the first row, gives

\det V(0,x_2, \cdots ,x_n)=x_2 \cdots x_n \det V(x_2, \cdots x_n).

Problem 1. Show that

\displaystyle \det V(x_1, \cdots ,x_n)=\prod_{n \ge i > j \ge 1}(x_i-x_j).

Solution. The proof is by induction on n. For n=2 we have \det V(x_1,x_2)=x_2-x_1. For the general case, first note that if one of the x_i‘s is zero, say x_1=0, then, by Remark 1,

\det V(x_1, \cdots ,x_n)=\det V(0,x_2, \cdots ,x_n)=x_2 \cdots x_n \det V(x_2, \cdots ,x_n)

and we are done by induction. Suppose now that x_i \ne 0 for all i. Consider the function

p(t):=\det V(t,x_2, \cdots ,x_n),

which is clearly a polynomial of degree at most n-1. It is also clear that p(x_2)= \cdots p(x_n)=0. Thus p(t)=c(t-x_2) \cdots (t-x_n), for some constant c. Now, using Remark 1,

(-1)^{n-1}cx_2 \cdots x_n=p(0)=\det V(0, x_2, \cdots , x_n)= x_2 \cdots x_n \det V(x_2, \cdots , x_n),

which gives c=(-1)^{n-1}\det V(x_2, \cdots , x_n). Hence

\begin{aligned}p(t)=(-1)^{n-1}(t-x_2) \cdots (t-x_n)\det V(x_2, \cdots , x_n)=(x_2-t) \cdots (x_n-t)\det V(x_2, \cdots , x_n)\end{aligned}

and so \det V(x_1, \cdots , x_n)=p(x_1)=(x_2-x_1) \cdots (x_n-x_1)\det V(x_2, \cdots , x_n). We are now done by induction. \ \Box

Remark 2. By Problem 1, V(x_1, \cdots , x_n) is invertible if and only if x_i \ne x_j for all i \ne j.

We now use Problem 1 to find the determinant of a slightly more complicated matrix.

Problem 2. Let A be the n \times n matrix whose (i,j)-entry is (x_i+y_j)^{n-1}. Show that

\displaystyle \det A=\prod_{k=1}^{n-1}\binom{n-1}{k}\prod_{n \ge i > j \ge 1}(x_i-x_j)(y_j-y_i).

Solution. Since

\displaystyle (x_i+y_j)^{n-1}=\sum_{k=0}^{n-1}\binom{n-1}{k}x_i^ky_j^{n-1-k},

we have A=BC, where

\displaystyle B=\begin{pmatrix}1 & \binom{n-1}{1}x_1 & \cdots & \binom{n-1}{n-1}x_1^{n-1} \\ 1 & \binom{n-1}{1}x_2 & \cdots & \binom{n-1}{n-1}x_2^{n-1} \\ \vdots & \vdots & \cdots & \vdots \\ 1 & \binom{n-1}{1}x_n & \cdots & \binom{n-1}{n-1}x_n^{n-1}\end{pmatrix}, \ \ \ \ \ \ C=\begin{pmatrix}y_1^{n-1} & y_2^{n-1} & \cdots & y_n^{n-1} \\ y_1^{n-2} & y_2^{n-2} & \cdots & y_n^{n-2} \\ \vdots & \vdots & \cdots & \vdots \\ 1 & 1 & \cdots & 1\end{pmatrix}.

Thus \det A=\det B \det C. Clearly

\displaystyle \det B=\prod_{k=1}^{n-1}\binom{n-1}{k}\det V(x_1, \cdots ,x_n)=\prod_{k=1}^{n-1}\binom{n-1}{k}\prod_{n \ge i > j \ge 1}(x_i-x_j),

by Problem 1. To find \det C, note that swapping columns of C^T, the transpose of C, gives

\displaystyle \det C=\det C^T=(-1)^{n(n-1)/2} \det V(y_1, \cdots , y_n)=\prod_{n \ge i > j \ge 1}(y_j-y_i),

by Problem 1. \ \Box

Definition. Given a vector \bold{v}=(c_0, c_1, \cdots ,c_{n-1}) \in \mathbb{C}^n, the circulant (or cyclic) matrix generated by \bold{v} is the n \times n matrix C(\bold{v}) defined as follows

C(\bold{v})=\begin{pmatrix}c_0 & c_1 & c_2 & \cdots & c_{n-2} & c_{n-1} \\ c_{n-1} &  c_0 & c_1 & \cdots & c_{n-3} & c_{n-2} \\ c_{n-2} & c_{n-1} & c_0 & \cdots & c_{n-4} & c_{n-3} \\ \vdots & \vdots & \vdots & \cdots & \vdots & \vdots \\ c_1 & c_2 & c_3 & \cdots & c_{n-1} & c_0 \end{pmatrix}.

Theorem. Let \bold{v}=(c_0, c_1, \cdots ,c_{n-1}) \in \mathbb{C}^n, and p(x)=c_0+c_1x + \cdots + c_{n-1}x^{n-1}. Let \alpha \in \mathbb{C} be such that \alpha^n=1, and let \alpha_k:=e^{2k\pi i/n}, \ 0 \le k \le n-1. Let \bold{x}(\alpha)=(1, \alpha, \alpha^2, \cdots , \alpha^{n-1})^T, where T means transpose.

i) C(\bold{v}) \bold{x}(\alpha)=p(\alpha)\bold{x}(\alpha).

ii) The vectors \bold{x}(\alpha_k), \ 0 \le k \le n-1, are \mathbb{C}-linearly independent.

iii) The eigenvalues of C(\bold{v}) are \lambda_k=p(\alpha_k), \ 0 \le k \le n-1, and \bold{x}(\alpha_k) is an eigenvector corresponding to \lambda_k. In particular, C(\bold{v}) is diagonalizable and

\displaystyle \det C(\bold{v})=\prod_{k=0}^{n-1}(c_0+c_1\alpha_k+ \cdots + c_{n-1}\alpha_k^{n-1}).

Proof. i) Using \alpha^n=1, we have

\begin{matrix} p(\alpha)=c_0+c_1\alpha + c_2\alpha^2+ \cdots +c_{n-2}\alpha^{n-2}+c_{n-1}\alpha^{n-1} \\ \\ \alpha p(\alpha)=c_{n-1} + c_0\alpha + c_1 \alpha^2 + \cdots +c_{n-3}\alpha^{n-2}+ c_{n-2}\alpha^{n-1}, \\ \\  \alpha^2p(\alpha)=c_{n-2}+c_{n-1}\alpha + c_0\alpha^2 + \cdots +c_{n-4}\alpha^{n-2}+ c_{n-3}\alpha^{n-1}, \\ \\ \vdots \\ \\ \alpha^{n-1}p(\alpha)=c_1+c_2\alpha+c_3\alpha^2+ \cdots + c_{n-1}\alpha^{n-2}+c_0\alpha^{n-1}.\end{matrix}

The matrix form of the above identities is

\begin{pmatrix}c_0 & c_1 & c_2 & \cdots & c_{n-2} & c_{n-1} \\ c_{n-1} &  c_0 & c_1 & \cdots & c_{n-3} & c_{n-2} \\ c_{n-2} & c_{n-1} & c_0 & \cdots & c_{n-4} & c_{n-3} \\ \vdots & \vdots & \vdots & \cdots & \vdots & \vdots \\ c_1 & c_2 & c_3 & \cdots & c_{n-1} & c_0 \end{pmatrix}\begin{pmatrix} 1 \\ \alpha \\ \alpha^2 \\ \vdots \\ \alpha^{n-1}\end{pmatrix}=p(\alpha)\begin{pmatrix} 1 \\ \alpha \\ \alpha^2 \\ \vdots \\ \alpha^{n-1}\end{pmatrix},

which is C(\bold{v}) \bold{x}(\alpha)=p(\alpha)\bold{x}(\alpha).

ii) Suppose that \sum_{k=0}^{n-1}z_k\bold{x}(\alpha_k)=0 for some z_k \in \mathbb{C}. Then \sum_{k=0}^{n-1}z_k\alpha_k^j=0 for all 0 \le j \le n-1, which, in matrix form, becomes

\begin{pmatrix}1 & 1 & \cdots & 1 \\ \alpha_0 & \alpha_1 & \cdots & \alpha_{n-1} \\ \vdots & \vdots & \cdots & \vdots \\ \alpha_0^{n-1} & \alpha_1^{n-1} & \cdots & \alpha_{n-1}^{n-1}\end{pmatrix} \begin{pmatrix}z_0 \\ z_1 \\ \vdots \\ z_{n-1}\end{pmatrix}=\begin{pmatrix}0 \\ 0 \\ \vdots \\ 0\end{pmatrix}.

Now, the n \times n matrix on the left-hand side of the above is a Vandermonde matrix which is invertible because \alpha_0, \cdots \alpha_{n-1} are pairwise distinct. Thus z_k=0 for all k, proving the linear independence.

iii) Since \alpha_k^n=1 for all 0 \le k \le n-1, we have C(\bold{v}) \bold{x}(\alpha_k)=p(\alpha_k)\bold{x}(\alpha_k), by i). Thus each \lambda_k=p(\alpha_k) is an eigenvalue of C(\bold{v}) with the corresponding eigenvector \bold{x}(\alpha_k). Since, by ii), \bold{x}(\alpha_k), \ 0 \le k \le n-1, are \mathbb{C}-linearly independent, \lambda_k, \ 0 \le k \le n-1, are all the eigenvalues of C(\bold{v}) and the matrix C(\bold{v}) is diagonalizable. Finally,

\displaystyle \det C(\bold{v})=\prod_{k=0}^{n-1}\lambda_k=\prod_{k=0}^{n-1}p(\alpha_k)=\prod_{k=0}^{n-1}(c_0+c_1\alpha_k + \cdots + c_{n-1}\alpha_k^{n-1}). \ \Box

Problem. Let n \ge 2 be an integer, and let f: \mathbb{R} \to \mathbb{R} be a function which is n times differentiable. Show that if \displaystyle \lim_{x\to\infty}f(x) is a finite number and \displaystyle \lim_{x\to\infty}f^{(n)}(x)=0, then \displaystyle \lim_{x\to\infty}f^{(k)}(x)=0 for all 1 \le k \le n-1. Here f^{(k)} is the kth derivative of f.

Solution. By Taylor’s theorem, for each x \in \mathbb{R} and 1 \le k \le n-1 there exists x_k \in (x,x+k) such that

\displaystyle \sum_{j=1}^{n-1}\frac{f^{(j)}(x)}{j!}k^j=f(x+k)-f(x)-\frac{f^{(n)}(x_k)}{n!}k^n,

which, in terms of matrices, can be written as

\displaystyle \begin{pmatrix}1 & 1 & \cdots & 1 \\ 2 & 2^2 & \cdots & 2^{n-1} \\ . & . & \cdots & . \\ . & . & \cdots & . \\ . & . & \cdots & . \\ n-1 & (n-1)^2 & \cdots & (n-1)^{n-1} \end{pmatrix}\begin{pmatrix}f'(x) \\ \frac{f''(x)}{2} \\ . \\ . \\ . \\ \frac{f^{(n-1)}(x)}{(n-1)!}\end{pmatrix}=\begin{pmatrix}g_1(x) \\ g_2(x) \\ . \\ . \\ . \\ g_{n-1}(x)\end{pmatrix}, \ \ \ \ \ \ \ \ (*)

where

\displaystyle g_j(x):=f(x+j)-f(x)-\frac{f^{(n)}(x_j)}{n!}j^n, \ \ \ \ \ 1 \le j \le n-1.

Since the square matrix on the left-hand side of (*) is invertible (Vandermonde), each f^{(k)}(x) is a linear combination of the functions g_1(x), \cdots , g_{n-1}(x). So, in order to prove that \displaystyle \lim_{x\to\infty}f^{(k)}(x)=0 for all 1 \le k \le n-1, we only need to show that \displaystyle \lim_{x\to\infty}g_j(x)=0 for all 1 \le j \le n-1. Well, for each j, we have \displaystyle \lim_{x\to\infty}(f(x+j)-f(x))=0 because we are given that \displaystyle \lim_{x\to\infty}f(x) is a finite number. We also have \displaystyle \lim_{x\to\infty}f^{(n)}(x_j)=0 because \displaystyle \lim_{x\to\infty}f^{(n)}(x)=0 and x_j \in (x, x+j). Thus \displaystyle \lim_{x\to\infty}g_i(x)=0 for all j. \ \Box